LSAT and Law School Admissions Forum

Get expert LSAT preparation and law school admissions advice from PowerScore Test Preparation.

 Administrator
PowerScore Staff
  • PowerScore Staff
  • Posts: 8919
  • Joined: Feb 02, 2011
|
#40886
Complete Question Explanation
(The complete setup for this game can be found here: lsat/viewtopic.php?t=15551)

The correct answer choice is (D)

This question suspends the second rule, and asks you to find an answer choice that could be substituted for that rule while retaining the exact same effects as the second rule.

Because the second rule involves two separate conditions, let’s consider each for a moment:

  • 1. ..... H is somewhere before K (H :longline: K)
    2. ..... H is not first
Thus, to create an acceptable substitute, we need an answer choice that addresses both aspects of the rule, and concomitantly does not add any additional conditions to the game. With these specifications in mind, let’s consider each answer choice:

Answer choice (A): Although this answer addresses the “H is not first” stipulation, it does not address the H :longline: K condition, and is therefore incorrect.

Answer choice (B): This answer choice addresses the H :longline: K stipulation by using G (which is linked to K through the first rule). However, this answer adds in the additional consideration that F or M is first, and we know from our initial discussion that F, I, L, or M could be first. Consequently, as this answer eliminates I or L from first, it adds a new effect and is incorrect.

Answer choice (C): This answer places H into the second, third, fourth, or fifth position. This satisfies the “H is not first” condition, but because K is not referenced (And thus it would be possible for the GK block to appear before H when H is one of the later positions), this answer also fails.

Answer choice (D): This answer addresses both conditions above and does not add any additional effects. Thus, this is the correct answer choice.

Answer choice (E): This answer satisfies both conditions in that H cannot be first and H :longline: K. Unfortunately, it also adds a new condition, namely that F must be first (F is first or seventh, and if F :longline: H, then F cannot be seventh). This additional condition is a new effect of the rule, meaning that this answer choice does not satisfy the “same effect” criterion established in the question stem.
 Sean22
  • Posts: 5
  • Joined: Oct 11, 2012
|
#6101
I apologize if this has been covered somewhere else, but I've noticed a question type in the games section that I haven't covered before. The questions ask you to select the answer choice that acts as a substitute for an already established rule. I noticed it on a previous test, but on this PT, Game 2: Question 10 stuck out. I'm not sure how to quickly and effectively solve this question and future ones like it. Any help is much appreciated!
User avatar
 Dave Killoran
PowerScore Staff
  • PowerScore Staff
  • Posts: 5850
  • Joined: Mar 25, 2011
|
#6111
Hey Sean,

We call those Substitution questions, and there was a post about that a while back. Here it is: http://forum.powerscore.com/lsat/viewto ... tion#p4253

Please check that out and let me know if it helps.

Thanks!
 Sean22
  • Posts: 5
  • Joined: Oct 11, 2012
|
#6125
Yes, this is what I was looking for. Thanks!
 moshei24
  • Posts: 465
  • Joined: Mar 20, 2012
|
#6395
Is the reason that (B) is wrong because it adds to the rule? As in it forces F or M to be first? It still forces H to be in 2-5, though.
 moshei24
  • Posts: 465
  • Joined: Mar 20, 2012
|
#6419
Thanks, Dave!
 SherryZ
  • Posts: 124
  • Joined: Oct 06, 2013
|
#13061
Hi there, thanks again for your help!

Dec 2009 LSAT, Sec 1 LG, Game 2, Q10:

I got this question correctly but it took me long time. Every time when I confront this type of "If REPLACE the original condition with one of the following NEW conditions, but other original conditions remain the same, which one will have the SAME EFFECT" questions, they are gonna take me long time to solve.

Could you tell me other methods to solve this type of question quickly instead of trying every single questions to prove whether it is right or wrong?

Thank you very much and looking forward to your reply!

---Sherry
User avatar
 KelseyWoods
PowerScore Staff
  • PowerScore Staff
  • Posts: 1079
  • Joined: Jun 26, 2013
|
#13072
Hi Sherry!

Unfortunately, these substitution questions are often difficult and time consuming and there aren't really any "shortcuts". In terms of strategy, a question like this is probably one to skip. That said, I have a few tips that might save you a little time.

First of all, I would recommend reading through all of the answer choices before trying any of them out. Sometimes you'll be able to eliminate one or two without testing it and sometimes you read answer choice (D) or (E) and it jumps out to you as the correct answer. So read them all first save yourself unnecessary work.

Pay attention to everything that the suspended rule does in the game and make sure the answer you pick has the exact same effects. They'll often throw in answer choices that satisfy one of the effects of the rule but don't have another effect. The more you understand the effects of the original rule, the better you'll be able to spot the identical effects in the substituted rule.

As always, beware of mistaken reversals and other shady conditional reasoning tricks they might try to throw at you.

Hope that helps!

Best,
Kelsey
 blairepstein
  • Posts: 1
  • Joined: May 25, 2016
|
#25765
I picked B for this one because I thought I would run out of time if I diagrammed each one so I went with my best guess. Now that I see why B is wrong, could you explain why E would be wrong? When I went back to look at the test I diagrammed E which says H-K so H-GK (just like the original rules) and F-H which means F can't go last so it goes first therefore H can't go first. Can you explain why this would be wrong?

Thank you!

Get the most out of your LSAT Prep Plus subscription.

Analyze and track your performance with our Testing and Analytics Package.